Proving Lim Sup Property for Bounded Sequences | Real Numbers HW

In summary, the logic seems to be ok. However, it is not guaranteed that all subsequences converge to the same limit, so you would need to know that the sequences are also monotonic in order to be sure.
  • #1
barksdalemc
55
0
I am trying to prove some things for a HW problem. Can you guys tell me if the following logic looks ok.

Let Un and Vn be bounded sequences of real numbers. If Un<=Vn for every n , show that lim sup n--->infinity Un <= lim sup n--->infinity Vn.

Here is what I wrote:

Let E1 and E2 be the set of all limits of subsequences of Un and Vn respectively. E1 and E2 are bounded and nonempty becuase Un and Vn are bounded, therefore E1 and E2 have a sup in R. Since all subsequences convege to the same limit as the sequence itself, the sup of E1 is the limit of Un and the limit of E2 is the limit of Vn. Therefore since Un<=Vn for every n the lim sup of Un<=lim supVn.
 
Physics news on Phys.org
  • #2
I made an error obviously. All subsequences do not converge to the limit of the sequence. Any hints?
 
  • #3
barksdalemc said:
I made an error obviously. All subsequences do not converge to the limit of the sequence. Any hints?
Your right. its not guaranteed that all subsequences converge to the same limit (or that they converge at all), you would need to know that the sequences are also monotonic which would imply they converge (and thus their limit is equal to their lim sup and lim inf)

If [itex](s_n)[/itex] is a sequence, then lim sup s_n is the limit of the sequence [itex] (\bar{s}_N)[/itex] whose N'th term (note the capitol N here) is [tex]sup\{s_n : n > N \}[/tex]

if Un <= Vn for all n, what does this imply about sup{Un : n > N} and sup{Vn : n > N} for each N?
 
  • #4
No matter what value Un takes on at each N, it will be less than Vn so the sup Un is less than or equal to sup Vn?
 
  • #5
barksdalemc said:
No matter what value Un takes on at each N, it will be less than Vn so the sup Un is less than or equal to sup Vn?
The notation "sup Un" makes no sense because Un is a real number not a set of real numbers, and sup only applies to sets.

You seem confused about what the limsup of a sequence is.

If [itex](s_n)[/itex] is a bounded sequence of real numbers, then we define

[tex] \limsup s_n = \lim_{k \rightarrow \infty}(sup \{ s_n : n > k \}) [/tex]

I used k here instead of N so as not to be confused with little n.

A way of thinking about this is that for each k we form the set {s_n : n > k}. Then define the sequence [itex](\bar{s}_k)[/itex] by,
[itex]\bar{s}_k = sup \{s_n : n > k \}[/itex]. Note that since (s_n) is a bounded sequence, {s_n : n > k} is a bounded set for each k, so the suprememum exists and is a real number. Thus [itex](\bar{s}_k)[/itex] is a sequence of real numbers indexed by k. Using this notation we then have,

[tex]\limsup s_n = \lim_{k \rightarrow \infty} \bar{s}_k [/tex]

Now, back to the problem. Your given that (U_n) and (V_n) are bounded sequences such that for every n, U_N <= V_n, and you need to show that limsup U_n <= limsup V_n.

Define the sequences [itex] (\bar{U}_k)[/itex] and [itex](\bar{V}_k)[/itex] the same way we did above for (s_n). Using this notation, what we need to show is,
[tex]\lim_{k \rightarrow \infty} \bar{U}_k \leq \lim_{k \rightarrow \infty} \bar{V}_k [/tex]

It is a known fact that if [itex](A_n)[/itex] and [itex](B_n)[/itex] are convergent sequences such that for each n, [itex]A_n \leq B_n[/itex], then [itex] \limA_n \leq \limB_n [/itex].

So if we can show that
1) [itex] (\bar{U}_k)[/itex] and [itex](\bar{V}_k)[/itex] converge, and
2) [itex](\bar{U}_k) \leq (\bar{V}_k)[/itex] for each k,
then we could use the above fact to get the desired conclusion.

Here are some hints on how to proceed:
to prove 1) show that [itex](\bar{U}_k)[/itex] and [itex](\bar{V}_k)[/itex] are bounded and decreasing.
Hint: if A and B are bounded sets of real numbers and [itex] A \subseteq B[/itex] then [itex] supA \leq supB[/itex].

to show 2) Try a proof by contradiction.

Does any of this make sense? If not which part(s) are confusing to you?
 
Last edited:

What is the limit supremum of a sequence?

The limit supremum of a sequence is the largest limit point of the sequence. In other words, it is the highest value that the sequence approaches as the number of terms in the sequence increases infinitely.

How do you find the limit supremum of a sequence?

To find the limit supremum of a sequence, you can take the supremum (or least upper bound) of the set of all limit points of the sequence. This can be done by finding the largest value in the set of limit points.

What is the difference between limit supremum and limit infimum?

The limit supremum is the largest limit point of a sequence, while the limit infimum is the smallest limit point. In other words, the limit supremum is the highest value that the sequence approaches, and the limit infimum is the lowest value that the sequence approaches.

Can a sequence have a limit supremum but not a limit?

Yes, it is possible for a sequence to have a limit supremum but not a limit. This means that the sequence has a highest limit point, but it does not necessarily converge to a single value. In this case, the limit supremum is often used as a measure of the "largest" value that the sequence approaches.

How is the limit supremum used in mathematical analysis?

The limit supremum is an important concept in mathematical analysis, particularly in the study of sequences and series. It is used to determine the convergence and divergence of a sequence, and to prove theorems about the behavior of mathematical functions. In many cases, it is also used to define important concepts such as continuity and compactness.

Similar threads

  • Calculus and Beyond Homework Help
Replies
8
Views
1K
  • Calculus and Beyond Homework Help
Replies
1
Views
259
  • Calculus and Beyond Homework Help
Replies
7
Views
1K
  • Calculus and Beyond Homework Help
Replies
1
Views
1K
  • Calculus and Beyond Homework Help
Replies
7
Views
2K
  • Calculus and Beyond Homework Help
Replies
8
Views
1K
  • Calculus and Beyond Homework Help
Replies
8
Views
2K
  • Calculus and Beyond Homework Help
Replies
3
Views
1K
  • Calculus and Beyond Homework Help
Replies
6
Views
2K
Replies
7
Views
2K
Back
Top